Difference between revisions of "2022 SSMO Team Round Problems/Problem 6"
(Created page with "==Problem== Let <math>n</math> be a positive integer, and let <math>x</math> be some variable. Define <math>P_{x,n}</math> as the maximum fraction of elements in the set of th...") |
(→Solution) |
||
Line 3: | Line 3: | ||
==Solution== | ==Solution== | ||
+ | |||
+ | <math>\bold{CLAIM}</math>: <math>P_n = \frac{n}{n+1}</math> | ||
+ | |||
+ | We maximize the number of elements in our subset by working from the highest elements to the lowest number of elements. Working through some numbers, we observe that the strategy is to first pick the top <math>\frac{n-1}{n}</math> numbers, and then skip the next <math>\frac{n-1}{n^2}</math> numbers, and then choose the next <math>\frac{n-1}{n^3}</math> numbers and so on. | ||
+ | |||
+ | For example, working through <math>P_{64,2}</math>, in order to maximize our set, we choose <math>(33,34,...,63,64)</math>, then <math>(9,10,...,15,16)</math>, then <math>(3,4)</math>, and lastly <math>1</math>. This follows the procedure above to maximize the number of elements in the set. | ||
+ | |||
+ | Therefore, we get <cmath>P_n = \frac{n-1}{n} + \frac{n-1}{n^3} + \frac{n-1}{n^5} + ...</cmath> which evaluates to our claim above. | ||
+ | |||
+ | From this claim, we have that <cmath>\prod_{n=2}^{100} P_n = \frac{2}{101}</cmath> so our answer is <math>2+101 = \boxed{103}</math> |
Revision as of 00:16, 18 February 2025
Problem
Let be a positive integer, and let
be some variable. Define
as the maximum fraction of elements in the set of the first
natural numbers that may be contained in a subset
such that if
is an element of
, then
is not. For example,
, since we take the set
. As
approaches infinity,
approaches a value
. Given that
where
and
are relatively prime positive integers, find
Solution
:
We maximize the number of elements in our subset by working from the highest elements to the lowest number of elements. Working through some numbers, we observe that the strategy is to first pick the top numbers, and then skip the next
numbers, and then choose the next
numbers and so on.
For example, working through , in order to maximize our set, we choose
, then
, then
, and lastly
. This follows the procedure above to maximize the number of elements in the set.
Therefore, we get which evaluates to our claim above.
From this claim, we have that so our answer is